Prove that $gcd(a^n - 1, a^m - 1) = a^{gcd(n, m)} - 1$$gcd(b^x - 1, b^y - 1, b^ z- 1,…) = b^{gcd(x, y,...

Do Legal Documents Require Signing In Standard Pen Colors?

Why do we read the Megillah by night and by day?

Why does the Sun have different day lengths, but not the gas giants?

Why is so much work done on numerical verification of the Riemann Hypothesis?

Creature in Shazam mid-credits scene?

Does an advisor owe his/her student anything? Will an advisor keep a PhD student only out of pity?

Melting point of aspirin, contradicting sources

Non-trope happy ending?

When a Cleric spontaneously casts a Cure Light Wounds spell, will a Pearl of Power recover the original spell or Cure Light Wounds?

Is there a single word describing earning money through any means?

Pre-mixing cryogenic fuels and using only one fuel tank

Travelling outside the UK without a passport

Why do compilers behave differently when static_cast(ing) a function to void*?

Does the expansion of the universe explain why the universe doesn't collapse?

What is Cash Advance APR?

Yosemite Fire Rings - What to Expect?

Closed-form expression for certain product

Removing files under particular conditions (number of files, file age)

What is the evidence for the "tyranny of the majority problem" in a direct democracy context?

What if a revenant (monster) gains fire resistance?

How could a planet have erratic days?

How can "mimic phobia" be cured or prevented?

Drawing ramified coverings with tikz

Should I stop contributing to retirement accounts?



Prove that $gcd(a^n - 1, a^m - 1) = a^{gcd(n, m)} - 1$


$gcd(b^x - 1, b^y - 1, b^ z- 1,…) = b^{gcd(x, y, z,…)} -1$Prove that if $d$ divides $n$, then $2^d -1$ divides $2^n -1$Gcd number theory proof: $(a^n-1,a^m-1)= a^{(m,n)}-1$Show that $gcd(2^m-1, 2^n-1) = 2^ {gcd(m,n)} -1$GCD of two big numbersProve: $gcd(n^a-1,n^b-1)=n^{gcd(a,b)}-1$Computing $gcd$ of very large numbers with powersGCD of two polynomials in Mod 2Prove $gcd(k, l) = d Rightarrow gcd(2^k - 1, 2^l - 1) = 2^d - 1$number theory division of power for the case $(n^r −1)$ divides $(n^m −1)$ if and only if $r$ divides $m$.Prove: $gcd(a,b) = gcd(a, b + at)$.How to prove that $zgcd(a,b)=gcd(za,zb)$Prove that if $gcd(a,b)=1$, then $gcd(acdot b,c) = gcd(a,c)cdot gcd(b,c)$.Prove that if $gcd(a,b)=1$ then $gcd(ab,c) = gcd(a,c) gcd(b,c)$Prove that $gcd(a, b) = gcd(a, b + ma)$?Prove that $gcd(a, b) = gcd(b, a-b)$Prove $gcd(a,b,c)=gcd(gcd(a,b),c)$.Suppose $gcd(a,y)=s$ and $gcd(b,y)=t$. Prove that $gcd(gcd(a,b),y)=gcd(s,t)$.Prove that$gcd(a+b, a-b) = gcd(2a, a-b) = gcd(a+b, 2b) $Prove that $gcd(a,b) = gcd (a+b, gcd(a,b))$













123












$begingroup$


For all $a, m, n in mathbb{Z}^+$,



$$gcd(a^n - 1, a^m - 1) = a^{gcd(n, m)} - 1$$










share|cite|improve this question











$endgroup$








  • 6




    $begingroup$
    Another question (math.stackexchange.com/questions/11567/…) was closed as a duplicate of this one where there is a second solution.
    $endgroup$
    – Qiaochu Yuan
    Dec 4 '10 at 14:17






  • 1




    $begingroup$
    Find here: Number Theory for Mathematical Contests, Example#245, Page#36.
    $endgroup$
    – lab bhattacharjee
    Jul 29 '12 at 16:56


















123












$begingroup$


For all $a, m, n in mathbb{Z}^+$,



$$gcd(a^n - 1, a^m - 1) = a^{gcd(n, m)} - 1$$










share|cite|improve this question











$endgroup$








  • 6




    $begingroup$
    Another question (math.stackexchange.com/questions/11567/…) was closed as a duplicate of this one where there is a second solution.
    $endgroup$
    – Qiaochu Yuan
    Dec 4 '10 at 14:17






  • 1




    $begingroup$
    Find here: Number Theory for Mathematical Contests, Example#245, Page#36.
    $endgroup$
    – lab bhattacharjee
    Jul 29 '12 at 16:56
















123












123








123


62



$begingroup$


For all $a, m, n in mathbb{Z}^+$,



$$gcd(a^n - 1, a^m - 1) = a^{gcd(n, m)} - 1$$










share|cite|improve this question











$endgroup$




For all $a, m, n in mathbb{Z}^+$,



$$gcd(a^n - 1, a^m - 1) = a^{gcd(n, m)} - 1$$







elementary-number-theory divisibility faq greatest-common-divisor






share|cite|improve this question















share|cite|improve this question













share|cite|improve this question




share|cite|improve this question








edited Jul 5 '15 at 11:43









Martin Sleziak

44.9k10122275




44.9k10122275










asked Oct 22 '10 at 0:47







Juan Liner















  • 6




    $begingroup$
    Another question (math.stackexchange.com/questions/11567/…) was closed as a duplicate of this one where there is a second solution.
    $endgroup$
    – Qiaochu Yuan
    Dec 4 '10 at 14:17






  • 1




    $begingroup$
    Find here: Number Theory for Mathematical Contests, Example#245, Page#36.
    $endgroup$
    – lab bhattacharjee
    Jul 29 '12 at 16:56
















  • 6




    $begingroup$
    Another question (math.stackexchange.com/questions/11567/…) was closed as a duplicate of this one where there is a second solution.
    $endgroup$
    – Qiaochu Yuan
    Dec 4 '10 at 14:17






  • 1




    $begingroup$
    Find here: Number Theory for Mathematical Contests, Example#245, Page#36.
    $endgroup$
    – lab bhattacharjee
    Jul 29 '12 at 16:56










6




6




$begingroup$
Another question (math.stackexchange.com/questions/11567/…) was closed as a duplicate of this one where there is a second solution.
$endgroup$
– Qiaochu Yuan
Dec 4 '10 at 14:17




$begingroup$
Another question (math.stackexchange.com/questions/11567/…) was closed as a duplicate of this one where there is a second solution.
$endgroup$
– Qiaochu Yuan
Dec 4 '10 at 14:17




1




1




$begingroup$
Find here: Number Theory for Mathematical Contests, Example#245, Page#36.
$endgroup$
– lab bhattacharjee
Jul 29 '12 at 16:56






$begingroup$
Find here: Number Theory for Mathematical Contests, Example#245, Page#36.
$endgroup$
– lab bhattacharjee
Jul 29 '12 at 16:56












7 Answers
7






active

oldest

votes


















63












$begingroup$

$rm f_{,n}: := a^n!-!1 = a^{n-m} : color{#c00}{(a^m!-!1)} + color{#0a0}{a^{n-m}!-!1}. $ Hence $rm: {f_{,n}! = color{#0a0}{f_{,n-m}}! + k color{#c00}{f_{,m}}},, kinmathbb Z.:$ Apply



Theorem $: $ If $rm f_{, n}: $ is an integer sequence with $rm f_{0} =, 0,: $ $rm :{ f_{,n}!equiv color{#0a0}{f_{,n-m}} (mod color{#c00}{f_{,m})}} $ for $rm: n > m, $ then $rm: (f_{,n},f_{,m}) = f_{,(n,:m)} : $ where $rm (i,:j) $ denotes $rm gcd(i,:j).:$



Proof $ $ By induction on $rm:n + m:$. The theorem is trivially true if $rm n = m $ or $rm n = 0 $ or $rm: m = 0.:$

So we may assume $rm:n > m > 0:$.$ $ Note $rm (f_{,n},f_{,m}) = (f_{,n-m},f_{,m}) $ follows from the hypothesis.

Since $rm (n-m)+m < n+m, $ induction yields $rm (f_{,n-m},f_{,m}), =, f_{,(n-m,:m)} =, f_{,(n,:m)}.$



See also this post for a conceptual proof exploiting the innate structure - an order ideal.






share|cite|improve this answer











$endgroup$













  • $begingroup$
    Sort of like the Fibonacci sequence!
    $endgroup$
    – cactus314
    May 23 '15 at 12:03










  • $begingroup$
    @john Yes, they are both strong divisibility sequences, i.e. $,(f_n,f_m) = f_{(n,m)}.,$ See here for the Fibonacci case.
    $endgroup$
    – Bill Dubuque
    May 23 '15 at 13:33





















34












$begingroup$

Below is a proof which has the neat feature that it immediately specializes
to a proof of the integer Bezout identity for $rm:x = 1,:$ allowing us to view it as a q-analog of the integer case.



E.g. for $rm m,n = 15,21$



$rmdisplaystylequadquadquadquadquadquadquad frac{x^3-1}{x-1} = (x^{15}! +! x^9! +! 1) frac{x^{15}!-!1}{x!-!1} - (x^9!+!x^3) frac{x^{21}!-!1}{x!-!1}$



for $rm x = 1 $ specializes to $ 3 = 3 (15) - 2 (21):, $ i.e. $rm (3) = (15,21) := gcd(15,21)$



Definition $rmdisplaystyle quad n' : := frac{x^n - 1}{x-1}:$. $quad$ Note $rmquad n' = n $ for $rm x = 1$.



Theorem $rmquad (m',n') = ((m,n)') $ for naturals $rm:m,n.$



Proof $ $ It is trivially true if $rm m = n $ or if $rm m = 0 $ or $rm n = 0.:$



W.l.o.g. suppose $rm:n > m > 0.:$ We proceed by induction on $rm:n! +! m.$



$begin{eqnarray}rm &rm x^n! -! 1 &=& rm x^r (x^m! -! 1) + x^r! -! 1 quad rm for r = n! -! m \
quadRightarrowquad &rmqquad n' &=& rm x^r m' + r' quad rm by dividing above by x!-!1 \
quadRightarrow &rm (m', n'), &=& rm (m', r') \
& &=&rm ((m,r)') quad by induction, applicable by: m!+!r = n < n!+!m \
& &=&rm ((m,n)') quad by r equiv n :(mod m)quad bf QED
end{eqnarray}$



Corollary $ $ Integer Bezout Theorem $ $ Proof: $ $ set $rm x = 1 $ above, i.e. erase primes.



A deeper understanding comes when one studies Divisibility Sequences
and Divisor Theory.






share|cite|improve this answer











$endgroup$













  • $begingroup$
    Is $((rm m,n)')$ supposed to be $((rm m,n))'$ i.e. $rm dfrac{x^{(m,n)}-1}{x-1}$?
    $endgroup$
    – Pedro Tamaroff
    Jun 18 '12 at 23:38












  • $begingroup$
    @Peter $ $ Let $rm:(m,n)' = dfrac{x^{,(m,n)}!-!1}{x!-!1} =: f.:$ Then $rm:((m,n)') = (f) = f:mathbb Z[x]:$ is a principal ideal, thus the equality $rm:(m',n') = ((m,n)'):$ denotes the ideal equality $rm:(g,h) = (f):$ for polynomials $rm:f,g,hinmathbb Z[x].:$ If you have no knowledge of ideals you can instead simply interpret it as saying that $rm:f:|:g,h:$ and $rm:f = a,g+b,h:$ for some $rm:a,bin mathbb Z[x],:$ which implies $rm:f = gcd(g,h).$
    $endgroup$
    – Bill Dubuque
    Jun 19 '12 at 0:17





















13












$begingroup$

Let $mge nge 1$. Apply Euclidean Algorithm.



$gcdleft(a^m-1,a^n-1right)=gcdleft(a^{n}left(a^{m-n}-1right),a^n-1right)$. Since $gcd(a^n,a^n-1)=1$, we get



$gcdleft(a^{m-n}-1,a^n-1right)$. Iterate this until it becomes $$gcdleft(a^{gcd(m,n)}-1,a^{gcd(m,n)}-1right)=a^{gcd(m,n)}-1$$






share|cite|improve this answer











$endgroup$













  • $begingroup$
    And this too is a duplicate of an answer in the 5-year-old linked duplicate thread.
    $endgroup$
    – Bill Dubuque
    Dec 31 '16 at 2:07





















8












$begingroup$

More generally, if $gcd(a,b)=1$, $a,b,m,ninmathbb Z^+$, $a> b$, then $$gcd(a^m-b^m,a^n-b^n)=a^{gcd(m,n)}-b^{gcd(m,n)}$$



Proof: Since $gcd(a,b)=1$, we get $gcd(b,d)=1$, so $b^{-1}bmod d$ exists.



$$dmid a^m-b^m, a^n-b^niff left(ab^{-1}right)^mequiv left(ab^{-1}right)^nequiv 1pmod{d}$$



$$iff text{ord}_{d}left(ab^{-1}right)mid m,niff text{ord}_{d}left(ab^{-1}right)mid gcd(m,n)$$



$$iff left(ab^{-1}right)^{gcd(m,n)}equiv 1pmod{d}iff a^{gcd(m,n)}equiv b^{gcd(m,n)}pmod{d}$$






share|cite|improve this answer











$endgroup$













  • $begingroup$
    This is precisely the homogenization $(a^n-1to a^n-b^n)$ of a proof in the 5-year-old duplicate thread linked in Yuan's comment on the question. To avoid posting such duplicate answers it's a good ides to first peruse duplicate links before posting an answer to a five year old question!
    $endgroup$
    – Bill Dubuque
    Dec 31 '16 at 2:03












  • $begingroup$
    Update: actually this homegenized version was posted 5 months prior in this answer.. There are probably older dupes too since this is a FAQ. Posting the link in case anyone decides to organize.
    $endgroup$
    – Bill Dubuque
    Jul 13 '17 at 20:44












  • $begingroup$
    I didn't understand why if gcd$(a,b) =1$ then gcd$(b, d) =1$? and why $left(ab^{-1}right)^mequiv left(ab^{-1}right)^nequiv 1pmod{d}$?
    $endgroup$
    – Vmimi
    Dec 2 '18 at 23:48



















8












$begingroup$

More generally, if $a,b,m,ninmathbb Z_{ge 1}$, $a>b$ and $(a,b)=1$ (as usual, $(a,b)$ denotes $gcd(a,b)$), then $$(a^m-b^m,a^n-b^n)=a^{(m,n)}-b^{(m,n)}$$



Proof: Use $,x^k-y^k=(x-y)(x^{k-1}+x^{k-2}y+cdots+xy^{k-2}+x^{k-1}),$



and use $nmid a,biff nmid (a,b)$ to prove:



$a^{(m,n)}-b^{(m,n)}mid a^m-b^m,, a^n-b^niff$



$a^{(m,n)}-b^{(m,n)}mid (a^m-b^m,a^n-b^n)=: d (1)$



$a^mequiv b^m,, a^nequiv b^n$ mod $d$ by definition of $d$.



Bezout's lemma gives $,mx+ny=(m,n),$ for some $x,yinBbb Z$.



$(a,b)=1iff (a,d)=(b,d)=1$, so $a^{mx},b^{ny}$ mod $d$ exist (notice $x,y$ can be negative).



$a^{mx}equiv b^{mx}$, $a^{ny}equiv b^{ny}$ mod $d$.



$a^{(m,n)}equiv a^{mx}a^{ny}equiv b^{mx}b^{ny}equiv b^{(m,n)}pmod{! d} (2)$



$(1)(2),Rightarrow, a^{(m,n)}-b^{(m,n)}=d$






share|cite|improve this answer











$endgroup$













  • $begingroup$
    What is $d$? I don't understand why gcd$(b, d)=1 $ and why do you need that to be true?
    $endgroup$
    – Vmimi
    Nov 30 '18 at 0:22





















5












$begingroup$

Let
$$gcd(a^n - 1, a^m - 1) = t$$
then
$$a^n equiv 1 ,big(text{ mod } tbig),quadtext{and}quad,a^m equiv 1 ,big(text{ mod } tbig)$$
And thus
$$a^{nx + my} equiv 1, big(text{ mod } tbig)$$
$forall,x,,yin mathbb{Z}$



According to the Extended Euclidean algorithm, we have
$$nx + my =gcd(n,m)$$
This follows
$$a^{nx + my} equiv 1 ,big(text{ mod } tbig) = a^{gcd(n,m)} equiv 1 big(text{ mod } tbig)impliesbig( a^{gcd(n,m)} - 1big) big| t$$



Therefore
$$a^{gcd(m,n)}-1, =gcd(a^m-1, a^n-1) $$






share|cite|improve this answer











$endgroup$





















    3












    $begingroup$

    Written for a duplicate question, this may be a bit more elementary than the other answers here, so I will post it:





    If $g=(a,b)$ and $G=left(p^a-1,p^b-1right)$, then
    $$
    left(p^g-1right)sum_{k=0}^{frac ag-1}p^{kg}=p^a-1
    $$
    and
    $$
    left(p^g-1right)sum_{k=0}^{frac bg-1}p^{kg}=p^b-1
    $$
    Thus, we have that
    $$
    left.p^g-1,middle|,Gright.
    $$





    For $xge0$,
    $$
    left(p^a-1right)sum_{k=0}^{x-1}p^{ak}=p^{ax}-1
    $$
    Therefore, we have that
    $$
    left.G,middle|,p^{ax}-1right.
    $$
    If $left.G,middle|,p^{ax-(b-1)y}-1right.$, then
    $$
    left(p^{ax-(b-1)y}-1right)-p^{ax-by}left(p^b-1right)=p^{ax-by}-1
    $$
    Therefore, for any $x,yge0$ so that $ax-byge0$,
    $$
    left.G,middle|,p^{ax-by}-1right.
    $$
    which means that
    $$
    left.G,middle|,p^g-1right.
    $$





    Putting all this together gives
    $$
    G=p^g-1
    $$






    share|cite|improve this answer











    $endgroup$













      Your Answer





      StackExchange.ifUsing("editor", function () {
      return StackExchange.using("mathjaxEditing", function () {
      StackExchange.MarkdownEditor.creationCallbacks.add(function (editor, postfix) {
      StackExchange.mathjaxEditing.prepareWmdForMathJax(editor, postfix, [["$", "$"], ["\\(","\\)"]]);
      });
      });
      }, "mathjax-editing");

      StackExchange.ready(function() {
      var channelOptions = {
      tags: "".split(" "),
      id: "69"
      };
      initTagRenderer("".split(" "), "".split(" "), channelOptions);

      StackExchange.using("externalEditor", function() {
      // Have to fire editor after snippets, if snippets enabled
      if (StackExchange.settings.snippets.snippetsEnabled) {
      StackExchange.using("snippets", function() {
      createEditor();
      });
      }
      else {
      createEditor();
      }
      });

      function createEditor() {
      StackExchange.prepareEditor({
      heartbeatType: 'answer',
      autoActivateHeartbeat: false,
      convertImagesToLinks: true,
      noModals: true,
      showLowRepImageUploadWarning: true,
      reputationToPostImages: 10,
      bindNavPrevention: true,
      postfix: "",
      imageUploader: {
      brandingHtml: "Powered by u003ca class="icon-imgur-white" href="https://imgur.com/"u003eu003c/au003e",
      contentPolicyHtml: "User contributions licensed under u003ca href="https://creativecommons.org/licenses/by-sa/3.0/"u003ecc by-sa 3.0 with attribution requiredu003c/au003e u003ca href="https://stackoverflow.com/legal/content-policy"u003e(content policy)u003c/au003e",
      allowUrls: true
      },
      noCode: true, onDemand: true,
      discardSelector: ".discard-answer"
      ,immediatelyShowMarkdownHelp:true
      });


      }
      });














      draft saved

      draft discarded


















      StackExchange.ready(
      function () {
      StackExchange.openid.initPostLogin('.new-post-login', 'https%3a%2f%2fmath.stackexchange.com%2fquestions%2f7473%2fprove-that-gcdan-1-am-1-a-gcdn-m-1%23new-answer', 'question_page');
      }
      );

      Post as a guest















      Required, but never shown
























      7 Answers
      7






      active

      oldest

      votes








      7 Answers
      7






      active

      oldest

      votes









      active

      oldest

      votes






      active

      oldest

      votes









      63












      $begingroup$

      $rm f_{,n}: := a^n!-!1 = a^{n-m} : color{#c00}{(a^m!-!1)} + color{#0a0}{a^{n-m}!-!1}. $ Hence $rm: {f_{,n}! = color{#0a0}{f_{,n-m}}! + k color{#c00}{f_{,m}}},, kinmathbb Z.:$ Apply



      Theorem $: $ If $rm f_{, n}: $ is an integer sequence with $rm f_{0} =, 0,: $ $rm :{ f_{,n}!equiv color{#0a0}{f_{,n-m}} (mod color{#c00}{f_{,m})}} $ for $rm: n > m, $ then $rm: (f_{,n},f_{,m}) = f_{,(n,:m)} : $ where $rm (i,:j) $ denotes $rm gcd(i,:j).:$



      Proof $ $ By induction on $rm:n + m:$. The theorem is trivially true if $rm n = m $ or $rm n = 0 $ or $rm: m = 0.:$

      So we may assume $rm:n > m > 0:$.$ $ Note $rm (f_{,n},f_{,m}) = (f_{,n-m},f_{,m}) $ follows from the hypothesis.

      Since $rm (n-m)+m < n+m, $ induction yields $rm (f_{,n-m},f_{,m}), =, f_{,(n-m,:m)} =, f_{,(n,:m)}.$



      See also this post for a conceptual proof exploiting the innate structure - an order ideal.






      share|cite|improve this answer











      $endgroup$













      • $begingroup$
        Sort of like the Fibonacci sequence!
        $endgroup$
        – cactus314
        May 23 '15 at 12:03










      • $begingroup$
        @john Yes, they are both strong divisibility sequences, i.e. $,(f_n,f_m) = f_{(n,m)}.,$ See here for the Fibonacci case.
        $endgroup$
        – Bill Dubuque
        May 23 '15 at 13:33


















      63












      $begingroup$

      $rm f_{,n}: := a^n!-!1 = a^{n-m} : color{#c00}{(a^m!-!1)} + color{#0a0}{a^{n-m}!-!1}. $ Hence $rm: {f_{,n}! = color{#0a0}{f_{,n-m}}! + k color{#c00}{f_{,m}}},, kinmathbb Z.:$ Apply



      Theorem $: $ If $rm f_{, n}: $ is an integer sequence with $rm f_{0} =, 0,: $ $rm :{ f_{,n}!equiv color{#0a0}{f_{,n-m}} (mod color{#c00}{f_{,m})}} $ for $rm: n > m, $ then $rm: (f_{,n},f_{,m}) = f_{,(n,:m)} : $ where $rm (i,:j) $ denotes $rm gcd(i,:j).:$



      Proof $ $ By induction on $rm:n + m:$. The theorem is trivially true if $rm n = m $ or $rm n = 0 $ or $rm: m = 0.:$

      So we may assume $rm:n > m > 0:$.$ $ Note $rm (f_{,n},f_{,m}) = (f_{,n-m},f_{,m}) $ follows from the hypothesis.

      Since $rm (n-m)+m < n+m, $ induction yields $rm (f_{,n-m},f_{,m}), =, f_{,(n-m,:m)} =, f_{,(n,:m)}.$



      See also this post for a conceptual proof exploiting the innate structure - an order ideal.






      share|cite|improve this answer











      $endgroup$













      • $begingroup$
        Sort of like the Fibonacci sequence!
        $endgroup$
        – cactus314
        May 23 '15 at 12:03










      • $begingroup$
        @john Yes, they are both strong divisibility sequences, i.e. $,(f_n,f_m) = f_{(n,m)}.,$ See here for the Fibonacci case.
        $endgroup$
        – Bill Dubuque
        May 23 '15 at 13:33
















      63












      63








      63





      $begingroup$

      $rm f_{,n}: := a^n!-!1 = a^{n-m} : color{#c00}{(a^m!-!1)} + color{#0a0}{a^{n-m}!-!1}. $ Hence $rm: {f_{,n}! = color{#0a0}{f_{,n-m}}! + k color{#c00}{f_{,m}}},, kinmathbb Z.:$ Apply



      Theorem $: $ If $rm f_{, n}: $ is an integer sequence with $rm f_{0} =, 0,: $ $rm :{ f_{,n}!equiv color{#0a0}{f_{,n-m}} (mod color{#c00}{f_{,m})}} $ for $rm: n > m, $ then $rm: (f_{,n},f_{,m}) = f_{,(n,:m)} : $ where $rm (i,:j) $ denotes $rm gcd(i,:j).:$



      Proof $ $ By induction on $rm:n + m:$. The theorem is trivially true if $rm n = m $ or $rm n = 0 $ or $rm: m = 0.:$

      So we may assume $rm:n > m > 0:$.$ $ Note $rm (f_{,n},f_{,m}) = (f_{,n-m},f_{,m}) $ follows from the hypothesis.

      Since $rm (n-m)+m < n+m, $ induction yields $rm (f_{,n-m},f_{,m}), =, f_{,(n-m,:m)} =, f_{,(n,:m)}.$



      See also this post for a conceptual proof exploiting the innate structure - an order ideal.






      share|cite|improve this answer











      $endgroup$



      $rm f_{,n}: := a^n!-!1 = a^{n-m} : color{#c00}{(a^m!-!1)} + color{#0a0}{a^{n-m}!-!1}. $ Hence $rm: {f_{,n}! = color{#0a0}{f_{,n-m}}! + k color{#c00}{f_{,m}}},, kinmathbb Z.:$ Apply



      Theorem $: $ If $rm f_{, n}: $ is an integer sequence with $rm f_{0} =, 0,: $ $rm :{ f_{,n}!equiv color{#0a0}{f_{,n-m}} (mod color{#c00}{f_{,m})}} $ for $rm: n > m, $ then $rm: (f_{,n},f_{,m}) = f_{,(n,:m)} : $ where $rm (i,:j) $ denotes $rm gcd(i,:j).:$



      Proof $ $ By induction on $rm:n + m:$. The theorem is trivially true if $rm n = m $ or $rm n = 0 $ or $rm: m = 0.:$

      So we may assume $rm:n > m > 0:$.$ $ Note $rm (f_{,n},f_{,m}) = (f_{,n-m},f_{,m}) $ follows from the hypothesis.

      Since $rm (n-m)+m < n+m, $ induction yields $rm (f_{,n-m},f_{,m}), =, f_{,(n-m,:m)} =, f_{,(n,:m)}.$



      See also this post for a conceptual proof exploiting the innate structure - an order ideal.







      share|cite|improve this answer














      share|cite|improve this answer



      share|cite|improve this answer








      edited Apr 13 '17 at 12:19









      Community

      1




      1










      answered Oct 23 '10 at 2:21









      Bill DubuqueBill Dubuque

      213k29195654




      213k29195654












      • $begingroup$
        Sort of like the Fibonacci sequence!
        $endgroup$
        – cactus314
        May 23 '15 at 12:03










      • $begingroup$
        @john Yes, they are both strong divisibility sequences, i.e. $,(f_n,f_m) = f_{(n,m)}.,$ See here for the Fibonacci case.
        $endgroup$
        – Bill Dubuque
        May 23 '15 at 13:33




















      • $begingroup$
        Sort of like the Fibonacci sequence!
        $endgroup$
        – cactus314
        May 23 '15 at 12:03










      • $begingroup$
        @john Yes, they are both strong divisibility sequences, i.e. $,(f_n,f_m) = f_{(n,m)}.,$ See here for the Fibonacci case.
        $endgroup$
        – Bill Dubuque
        May 23 '15 at 13:33


















      $begingroup$
      Sort of like the Fibonacci sequence!
      $endgroup$
      – cactus314
      May 23 '15 at 12:03




      $begingroup$
      Sort of like the Fibonacci sequence!
      $endgroup$
      – cactus314
      May 23 '15 at 12:03












      $begingroup$
      @john Yes, they are both strong divisibility sequences, i.e. $,(f_n,f_m) = f_{(n,m)}.,$ See here for the Fibonacci case.
      $endgroup$
      – Bill Dubuque
      May 23 '15 at 13:33






      $begingroup$
      @john Yes, they are both strong divisibility sequences, i.e. $,(f_n,f_m) = f_{(n,m)}.,$ See here for the Fibonacci case.
      $endgroup$
      – Bill Dubuque
      May 23 '15 at 13:33













      34












      $begingroup$

      Below is a proof which has the neat feature that it immediately specializes
      to a proof of the integer Bezout identity for $rm:x = 1,:$ allowing us to view it as a q-analog of the integer case.



      E.g. for $rm m,n = 15,21$



      $rmdisplaystylequadquadquadquadquadquadquad frac{x^3-1}{x-1} = (x^{15}! +! x^9! +! 1) frac{x^{15}!-!1}{x!-!1} - (x^9!+!x^3) frac{x^{21}!-!1}{x!-!1}$



      for $rm x = 1 $ specializes to $ 3 = 3 (15) - 2 (21):, $ i.e. $rm (3) = (15,21) := gcd(15,21)$



      Definition $rmdisplaystyle quad n' : := frac{x^n - 1}{x-1}:$. $quad$ Note $rmquad n' = n $ for $rm x = 1$.



      Theorem $rmquad (m',n') = ((m,n)') $ for naturals $rm:m,n.$



      Proof $ $ It is trivially true if $rm m = n $ or if $rm m = 0 $ or $rm n = 0.:$



      W.l.o.g. suppose $rm:n > m > 0.:$ We proceed by induction on $rm:n! +! m.$



      $begin{eqnarray}rm &rm x^n! -! 1 &=& rm x^r (x^m! -! 1) + x^r! -! 1 quad rm for r = n! -! m \
      quadRightarrowquad &rmqquad n' &=& rm x^r m' + r' quad rm by dividing above by x!-!1 \
      quadRightarrow &rm (m', n'), &=& rm (m', r') \
      & &=&rm ((m,r)') quad by induction, applicable by: m!+!r = n < n!+!m \
      & &=&rm ((m,n)') quad by r equiv n :(mod m)quad bf QED
      end{eqnarray}$



      Corollary $ $ Integer Bezout Theorem $ $ Proof: $ $ set $rm x = 1 $ above, i.e. erase primes.



      A deeper understanding comes when one studies Divisibility Sequences
      and Divisor Theory.






      share|cite|improve this answer











      $endgroup$













      • $begingroup$
        Is $((rm m,n)')$ supposed to be $((rm m,n))'$ i.e. $rm dfrac{x^{(m,n)}-1}{x-1}$?
        $endgroup$
        – Pedro Tamaroff
        Jun 18 '12 at 23:38












      • $begingroup$
        @Peter $ $ Let $rm:(m,n)' = dfrac{x^{,(m,n)}!-!1}{x!-!1} =: f.:$ Then $rm:((m,n)') = (f) = f:mathbb Z[x]:$ is a principal ideal, thus the equality $rm:(m',n') = ((m,n)'):$ denotes the ideal equality $rm:(g,h) = (f):$ for polynomials $rm:f,g,hinmathbb Z[x].:$ If you have no knowledge of ideals you can instead simply interpret it as saying that $rm:f:|:g,h:$ and $rm:f = a,g+b,h:$ for some $rm:a,bin mathbb Z[x],:$ which implies $rm:f = gcd(g,h).$
        $endgroup$
        – Bill Dubuque
        Jun 19 '12 at 0:17


















      34












      $begingroup$

      Below is a proof which has the neat feature that it immediately specializes
      to a proof of the integer Bezout identity for $rm:x = 1,:$ allowing us to view it as a q-analog of the integer case.



      E.g. for $rm m,n = 15,21$



      $rmdisplaystylequadquadquadquadquadquadquad frac{x^3-1}{x-1} = (x^{15}! +! x^9! +! 1) frac{x^{15}!-!1}{x!-!1} - (x^9!+!x^3) frac{x^{21}!-!1}{x!-!1}$



      for $rm x = 1 $ specializes to $ 3 = 3 (15) - 2 (21):, $ i.e. $rm (3) = (15,21) := gcd(15,21)$



      Definition $rmdisplaystyle quad n' : := frac{x^n - 1}{x-1}:$. $quad$ Note $rmquad n' = n $ for $rm x = 1$.



      Theorem $rmquad (m',n') = ((m,n)') $ for naturals $rm:m,n.$



      Proof $ $ It is trivially true if $rm m = n $ or if $rm m = 0 $ or $rm n = 0.:$



      W.l.o.g. suppose $rm:n > m > 0.:$ We proceed by induction on $rm:n! +! m.$



      $begin{eqnarray}rm &rm x^n! -! 1 &=& rm x^r (x^m! -! 1) + x^r! -! 1 quad rm for r = n! -! m \
      quadRightarrowquad &rmqquad n' &=& rm x^r m' + r' quad rm by dividing above by x!-!1 \
      quadRightarrow &rm (m', n'), &=& rm (m', r') \
      & &=&rm ((m,r)') quad by induction, applicable by: m!+!r = n < n!+!m \
      & &=&rm ((m,n)') quad by r equiv n :(mod m)quad bf QED
      end{eqnarray}$



      Corollary $ $ Integer Bezout Theorem $ $ Proof: $ $ set $rm x = 1 $ above, i.e. erase primes.



      A deeper understanding comes when one studies Divisibility Sequences
      and Divisor Theory.






      share|cite|improve this answer











      $endgroup$













      • $begingroup$
        Is $((rm m,n)')$ supposed to be $((rm m,n))'$ i.e. $rm dfrac{x^{(m,n)}-1}{x-1}$?
        $endgroup$
        – Pedro Tamaroff
        Jun 18 '12 at 23:38












      • $begingroup$
        @Peter $ $ Let $rm:(m,n)' = dfrac{x^{,(m,n)}!-!1}{x!-!1} =: f.:$ Then $rm:((m,n)') = (f) = f:mathbb Z[x]:$ is a principal ideal, thus the equality $rm:(m',n') = ((m,n)'):$ denotes the ideal equality $rm:(g,h) = (f):$ for polynomials $rm:f,g,hinmathbb Z[x].:$ If you have no knowledge of ideals you can instead simply interpret it as saying that $rm:f:|:g,h:$ and $rm:f = a,g+b,h:$ for some $rm:a,bin mathbb Z[x],:$ which implies $rm:f = gcd(g,h).$
        $endgroup$
        – Bill Dubuque
        Jun 19 '12 at 0:17
















      34












      34








      34





      $begingroup$

      Below is a proof which has the neat feature that it immediately specializes
      to a proof of the integer Bezout identity for $rm:x = 1,:$ allowing us to view it as a q-analog of the integer case.



      E.g. for $rm m,n = 15,21$



      $rmdisplaystylequadquadquadquadquadquadquad frac{x^3-1}{x-1} = (x^{15}! +! x^9! +! 1) frac{x^{15}!-!1}{x!-!1} - (x^9!+!x^3) frac{x^{21}!-!1}{x!-!1}$



      for $rm x = 1 $ specializes to $ 3 = 3 (15) - 2 (21):, $ i.e. $rm (3) = (15,21) := gcd(15,21)$



      Definition $rmdisplaystyle quad n' : := frac{x^n - 1}{x-1}:$. $quad$ Note $rmquad n' = n $ for $rm x = 1$.



      Theorem $rmquad (m',n') = ((m,n)') $ for naturals $rm:m,n.$



      Proof $ $ It is trivially true if $rm m = n $ or if $rm m = 0 $ or $rm n = 0.:$



      W.l.o.g. suppose $rm:n > m > 0.:$ We proceed by induction on $rm:n! +! m.$



      $begin{eqnarray}rm &rm x^n! -! 1 &=& rm x^r (x^m! -! 1) + x^r! -! 1 quad rm for r = n! -! m \
      quadRightarrowquad &rmqquad n' &=& rm x^r m' + r' quad rm by dividing above by x!-!1 \
      quadRightarrow &rm (m', n'), &=& rm (m', r') \
      & &=&rm ((m,r)') quad by induction, applicable by: m!+!r = n < n!+!m \
      & &=&rm ((m,n)') quad by r equiv n :(mod m)quad bf QED
      end{eqnarray}$



      Corollary $ $ Integer Bezout Theorem $ $ Proof: $ $ set $rm x = 1 $ above, i.e. erase primes.



      A deeper understanding comes when one studies Divisibility Sequences
      and Divisor Theory.






      share|cite|improve this answer











      $endgroup$



      Below is a proof which has the neat feature that it immediately specializes
      to a proof of the integer Bezout identity for $rm:x = 1,:$ allowing us to view it as a q-analog of the integer case.



      E.g. for $rm m,n = 15,21$



      $rmdisplaystylequadquadquadquadquadquadquad frac{x^3-1}{x-1} = (x^{15}! +! x^9! +! 1) frac{x^{15}!-!1}{x!-!1} - (x^9!+!x^3) frac{x^{21}!-!1}{x!-!1}$



      for $rm x = 1 $ specializes to $ 3 = 3 (15) - 2 (21):, $ i.e. $rm (3) = (15,21) := gcd(15,21)$



      Definition $rmdisplaystyle quad n' : := frac{x^n - 1}{x-1}:$. $quad$ Note $rmquad n' = n $ for $rm x = 1$.



      Theorem $rmquad (m',n') = ((m,n)') $ for naturals $rm:m,n.$



      Proof $ $ It is trivially true if $rm m = n $ or if $rm m = 0 $ or $rm n = 0.:$



      W.l.o.g. suppose $rm:n > m > 0.:$ We proceed by induction on $rm:n! +! m.$



      $begin{eqnarray}rm &rm x^n! -! 1 &=& rm x^r (x^m! -! 1) + x^r! -! 1 quad rm for r = n! -! m \
      quadRightarrowquad &rmqquad n' &=& rm x^r m' + r' quad rm by dividing above by x!-!1 \
      quadRightarrow &rm (m', n'), &=& rm (m', r') \
      & &=&rm ((m,r)') quad by induction, applicable by: m!+!r = n < n!+!m \
      & &=&rm ((m,n)') quad by r equiv n :(mod m)quad bf QED
      end{eqnarray}$



      Corollary $ $ Integer Bezout Theorem $ $ Proof: $ $ set $rm x = 1 $ above, i.e. erase primes.



      A deeper understanding comes when one studies Divisibility Sequences
      and Divisor Theory.







      share|cite|improve this answer














      share|cite|improve this answer



      share|cite|improve this answer








      edited Jul 4 '15 at 19:25

























      answered Oct 22 '10 at 1:51









      Bill DubuqueBill Dubuque

      213k29195654




      213k29195654












      • $begingroup$
        Is $((rm m,n)')$ supposed to be $((rm m,n))'$ i.e. $rm dfrac{x^{(m,n)}-1}{x-1}$?
        $endgroup$
        – Pedro Tamaroff
        Jun 18 '12 at 23:38












      • $begingroup$
        @Peter $ $ Let $rm:(m,n)' = dfrac{x^{,(m,n)}!-!1}{x!-!1} =: f.:$ Then $rm:((m,n)') = (f) = f:mathbb Z[x]:$ is a principal ideal, thus the equality $rm:(m',n') = ((m,n)'):$ denotes the ideal equality $rm:(g,h) = (f):$ for polynomials $rm:f,g,hinmathbb Z[x].:$ If you have no knowledge of ideals you can instead simply interpret it as saying that $rm:f:|:g,h:$ and $rm:f = a,g+b,h:$ for some $rm:a,bin mathbb Z[x],:$ which implies $rm:f = gcd(g,h).$
        $endgroup$
        – Bill Dubuque
        Jun 19 '12 at 0:17




















      • $begingroup$
        Is $((rm m,n)')$ supposed to be $((rm m,n))'$ i.e. $rm dfrac{x^{(m,n)}-1}{x-1}$?
        $endgroup$
        – Pedro Tamaroff
        Jun 18 '12 at 23:38












      • $begingroup$
        @Peter $ $ Let $rm:(m,n)' = dfrac{x^{,(m,n)}!-!1}{x!-!1} =: f.:$ Then $rm:((m,n)') = (f) = f:mathbb Z[x]:$ is a principal ideal, thus the equality $rm:(m',n') = ((m,n)'):$ denotes the ideal equality $rm:(g,h) = (f):$ for polynomials $rm:f,g,hinmathbb Z[x].:$ If you have no knowledge of ideals you can instead simply interpret it as saying that $rm:f:|:g,h:$ and $rm:f = a,g+b,h:$ for some $rm:a,bin mathbb Z[x],:$ which implies $rm:f = gcd(g,h).$
        $endgroup$
        – Bill Dubuque
        Jun 19 '12 at 0:17


















      $begingroup$
      Is $((rm m,n)')$ supposed to be $((rm m,n))'$ i.e. $rm dfrac{x^{(m,n)}-1}{x-1}$?
      $endgroup$
      – Pedro Tamaroff
      Jun 18 '12 at 23:38






      $begingroup$
      Is $((rm m,n)')$ supposed to be $((rm m,n))'$ i.e. $rm dfrac{x^{(m,n)}-1}{x-1}$?
      $endgroup$
      – Pedro Tamaroff
      Jun 18 '12 at 23:38














      $begingroup$
      @Peter $ $ Let $rm:(m,n)' = dfrac{x^{,(m,n)}!-!1}{x!-!1} =: f.:$ Then $rm:((m,n)') = (f) = f:mathbb Z[x]:$ is a principal ideal, thus the equality $rm:(m',n') = ((m,n)'):$ denotes the ideal equality $rm:(g,h) = (f):$ for polynomials $rm:f,g,hinmathbb Z[x].:$ If you have no knowledge of ideals you can instead simply interpret it as saying that $rm:f:|:g,h:$ and $rm:f = a,g+b,h:$ for some $rm:a,bin mathbb Z[x],:$ which implies $rm:f = gcd(g,h).$
      $endgroup$
      – Bill Dubuque
      Jun 19 '12 at 0:17






      $begingroup$
      @Peter $ $ Let $rm:(m,n)' = dfrac{x^{,(m,n)}!-!1}{x!-!1} =: f.:$ Then $rm:((m,n)') = (f) = f:mathbb Z[x]:$ is a principal ideal, thus the equality $rm:(m',n') = ((m,n)'):$ denotes the ideal equality $rm:(g,h) = (f):$ for polynomials $rm:f,g,hinmathbb Z[x].:$ If you have no knowledge of ideals you can instead simply interpret it as saying that $rm:f:|:g,h:$ and $rm:f = a,g+b,h:$ for some $rm:a,bin mathbb Z[x],:$ which implies $rm:f = gcd(g,h).$
      $endgroup$
      – Bill Dubuque
      Jun 19 '12 at 0:17













      13












      $begingroup$

      Let $mge nge 1$. Apply Euclidean Algorithm.



      $gcdleft(a^m-1,a^n-1right)=gcdleft(a^{n}left(a^{m-n}-1right),a^n-1right)$. Since $gcd(a^n,a^n-1)=1$, we get



      $gcdleft(a^{m-n}-1,a^n-1right)$. Iterate this until it becomes $$gcdleft(a^{gcd(m,n)}-1,a^{gcd(m,n)}-1right)=a^{gcd(m,n)}-1$$






      share|cite|improve this answer











      $endgroup$













      • $begingroup$
        And this too is a duplicate of an answer in the 5-year-old linked duplicate thread.
        $endgroup$
        – Bill Dubuque
        Dec 31 '16 at 2:07


















      13












      $begingroup$

      Let $mge nge 1$. Apply Euclidean Algorithm.



      $gcdleft(a^m-1,a^n-1right)=gcdleft(a^{n}left(a^{m-n}-1right),a^n-1right)$. Since $gcd(a^n,a^n-1)=1$, we get



      $gcdleft(a^{m-n}-1,a^n-1right)$. Iterate this until it becomes $$gcdleft(a^{gcd(m,n)}-1,a^{gcd(m,n)}-1right)=a^{gcd(m,n)}-1$$






      share|cite|improve this answer











      $endgroup$













      • $begingroup$
        And this too is a duplicate of an answer in the 5-year-old linked duplicate thread.
        $endgroup$
        – Bill Dubuque
        Dec 31 '16 at 2:07
















      13












      13








      13





      $begingroup$

      Let $mge nge 1$. Apply Euclidean Algorithm.



      $gcdleft(a^m-1,a^n-1right)=gcdleft(a^{n}left(a^{m-n}-1right),a^n-1right)$. Since $gcd(a^n,a^n-1)=1$, we get



      $gcdleft(a^{m-n}-1,a^n-1right)$. Iterate this until it becomes $$gcdleft(a^{gcd(m,n)}-1,a^{gcd(m,n)}-1right)=a^{gcd(m,n)}-1$$






      share|cite|improve this answer











      $endgroup$



      Let $mge nge 1$. Apply Euclidean Algorithm.



      $gcdleft(a^m-1,a^n-1right)=gcdleft(a^{n}left(a^{m-n}-1right),a^n-1right)$. Since $gcd(a^n,a^n-1)=1$, we get



      $gcdleft(a^{m-n}-1,a^n-1right)$. Iterate this until it becomes $$gcdleft(a^{gcd(m,n)}-1,a^{gcd(m,n)}-1right)=a^{gcd(m,n)}-1$$







      share|cite|improve this answer














      share|cite|improve this answer



      share|cite|improve this answer








      edited Jan 8 '16 at 18:55

























      answered Oct 25 '15 at 10:23









      user236182user236182

      12k11333




      12k11333












      • $begingroup$
        And this too is a duplicate of an answer in the 5-year-old linked duplicate thread.
        $endgroup$
        – Bill Dubuque
        Dec 31 '16 at 2:07




















      • $begingroup$
        And this too is a duplicate of an answer in the 5-year-old linked duplicate thread.
        $endgroup$
        – Bill Dubuque
        Dec 31 '16 at 2:07


















      $begingroup$
      And this too is a duplicate of an answer in the 5-year-old linked duplicate thread.
      $endgroup$
      – Bill Dubuque
      Dec 31 '16 at 2:07






      $begingroup$
      And this too is a duplicate of an answer in the 5-year-old linked duplicate thread.
      $endgroup$
      – Bill Dubuque
      Dec 31 '16 at 2:07













      8












      $begingroup$

      More generally, if $gcd(a,b)=1$, $a,b,m,ninmathbb Z^+$, $a> b$, then $$gcd(a^m-b^m,a^n-b^n)=a^{gcd(m,n)}-b^{gcd(m,n)}$$



      Proof: Since $gcd(a,b)=1$, we get $gcd(b,d)=1$, so $b^{-1}bmod d$ exists.



      $$dmid a^m-b^m, a^n-b^niff left(ab^{-1}right)^mequiv left(ab^{-1}right)^nequiv 1pmod{d}$$



      $$iff text{ord}_{d}left(ab^{-1}right)mid m,niff text{ord}_{d}left(ab^{-1}right)mid gcd(m,n)$$



      $$iff left(ab^{-1}right)^{gcd(m,n)}equiv 1pmod{d}iff a^{gcd(m,n)}equiv b^{gcd(m,n)}pmod{d}$$






      share|cite|improve this answer











      $endgroup$













      • $begingroup$
        This is precisely the homogenization $(a^n-1to a^n-b^n)$ of a proof in the 5-year-old duplicate thread linked in Yuan's comment on the question. To avoid posting such duplicate answers it's a good ides to first peruse duplicate links before posting an answer to a five year old question!
        $endgroup$
        – Bill Dubuque
        Dec 31 '16 at 2:03












      • $begingroup$
        Update: actually this homegenized version was posted 5 months prior in this answer.. There are probably older dupes too since this is a FAQ. Posting the link in case anyone decides to organize.
        $endgroup$
        – Bill Dubuque
        Jul 13 '17 at 20:44












      • $begingroup$
        I didn't understand why if gcd$(a,b) =1$ then gcd$(b, d) =1$? and why $left(ab^{-1}right)^mequiv left(ab^{-1}right)^nequiv 1pmod{d}$?
        $endgroup$
        – Vmimi
        Dec 2 '18 at 23:48
















      8












      $begingroup$

      More generally, if $gcd(a,b)=1$, $a,b,m,ninmathbb Z^+$, $a> b$, then $$gcd(a^m-b^m,a^n-b^n)=a^{gcd(m,n)}-b^{gcd(m,n)}$$



      Proof: Since $gcd(a,b)=1$, we get $gcd(b,d)=1$, so $b^{-1}bmod d$ exists.



      $$dmid a^m-b^m, a^n-b^niff left(ab^{-1}right)^mequiv left(ab^{-1}right)^nequiv 1pmod{d}$$



      $$iff text{ord}_{d}left(ab^{-1}right)mid m,niff text{ord}_{d}left(ab^{-1}right)mid gcd(m,n)$$



      $$iff left(ab^{-1}right)^{gcd(m,n)}equiv 1pmod{d}iff a^{gcd(m,n)}equiv b^{gcd(m,n)}pmod{d}$$






      share|cite|improve this answer











      $endgroup$













      • $begingroup$
        This is precisely the homogenization $(a^n-1to a^n-b^n)$ of a proof in the 5-year-old duplicate thread linked in Yuan's comment on the question. To avoid posting such duplicate answers it's a good ides to first peruse duplicate links before posting an answer to a five year old question!
        $endgroup$
        – Bill Dubuque
        Dec 31 '16 at 2:03












      • $begingroup$
        Update: actually this homegenized version was posted 5 months prior in this answer.. There are probably older dupes too since this is a FAQ. Posting the link in case anyone decides to organize.
        $endgroup$
        – Bill Dubuque
        Jul 13 '17 at 20:44












      • $begingroup$
        I didn't understand why if gcd$(a,b) =1$ then gcd$(b, d) =1$? and why $left(ab^{-1}right)^mequiv left(ab^{-1}right)^nequiv 1pmod{d}$?
        $endgroup$
        – Vmimi
        Dec 2 '18 at 23:48














      8












      8








      8





      $begingroup$

      More generally, if $gcd(a,b)=1$, $a,b,m,ninmathbb Z^+$, $a> b$, then $$gcd(a^m-b^m,a^n-b^n)=a^{gcd(m,n)}-b^{gcd(m,n)}$$



      Proof: Since $gcd(a,b)=1$, we get $gcd(b,d)=1$, so $b^{-1}bmod d$ exists.



      $$dmid a^m-b^m, a^n-b^niff left(ab^{-1}right)^mequiv left(ab^{-1}right)^nequiv 1pmod{d}$$



      $$iff text{ord}_{d}left(ab^{-1}right)mid m,niff text{ord}_{d}left(ab^{-1}right)mid gcd(m,n)$$



      $$iff left(ab^{-1}right)^{gcd(m,n)}equiv 1pmod{d}iff a^{gcd(m,n)}equiv b^{gcd(m,n)}pmod{d}$$






      share|cite|improve this answer











      $endgroup$



      More generally, if $gcd(a,b)=1$, $a,b,m,ninmathbb Z^+$, $a> b$, then $$gcd(a^m-b^m,a^n-b^n)=a^{gcd(m,n)}-b^{gcd(m,n)}$$



      Proof: Since $gcd(a,b)=1$, we get $gcd(b,d)=1$, so $b^{-1}bmod d$ exists.



      $$dmid a^m-b^m, a^n-b^niff left(ab^{-1}right)^mequiv left(ab^{-1}right)^nequiv 1pmod{d}$$



      $$iff text{ord}_{d}left(ab^{-1}right)mid m,niff text{ord}_{d}left(ab^{-1}right)mid gcd(m,n)$$



      $$iff left(ab^{-1}right)^{gcd(m,n)}equiv 1pmod{d}iff a^{gcd(m,n)}equiv b^{gcd(m,n)}pmod{d}$$







      share|cite|improve this answer














      share|cite|improve this answer



      share|cite|improve this answer








      edited Dec 1 '17 at 10:31

























      answered Oct 4 '15 at 17:03









      user236182user236182

      12k11333




      12k11333












      • $begingroup$
        This is precisely the homogenization $(a^n-1to a^n-b^n)$ of a proof in the 5-year-old duplicate thread linked in Yuan's comment on the question. To avoid posting such duplicate answers it's a good ides to first peruse duplicate links before posting an answer to a five year old question!
        $endgroup$
        – Bill Dubuque
        Dec 31 '16 at 2:03












      • $begingroup$
        Update: actually this homegenized version was posted 5 months prior in this answer.. There are probably older dupes too since this is a FAQ. Posting the link in case anyone decides to organize.
        $endgroup$
        – Bill Dubuque
        Jul 13 '17 at 20:44












      • $begingroup$
        I didn't understand why if gcd$(a,b) =1$ then gcd$(b, d) =1$? and why $left(ab^{-1}right)^mequiv left(ab^{-1}right)^nequiv 1pmod{d}$?
        $endgroup$
        – Vmimi
        Dec 2 '18 at 23:48


















      • $begingroup$
        This is precisely the homogenization $(a^n-1to a^n-b^n)$ of a proof in the 5-year-old duplicate thread linked in Yuan's comment on the question. To avoid posting such duplicate answers it's a good ides to first peruse duplicate links before posting an answer to a five year old question!
        $endgroup$
        – Bill Dubuque
        Dec 31 '16 at 2:03












      • $begingroup$
        Update: actually this homegenized version was posted 5 months prior in this answer.. There are probably older dupes too since this is a FAQ. Posting the link in case anyone decides to organize.
        $endgroup$
        – Bill Dubuque
        Jul 13 '17 at 20:44












      • $begingroup$
        I didn't understand why if gcd$(a,b) =1$ then gcd$(b, d) =1$? and why $left(ab^{-1}right)^mequiv left(ab^{-1}right)^nequiv 1pmod{d}$?
        $endgroup$
        – Vmimi
        Dec 2 '18 at 23:48
















      $begingroup$
      This is precisely the homogenization $(a^n-1to a^n-b^n)$ of a proof in the 5-year-old duplicate thread linked in Yuan's comment on the question. To avoid posting such duplicate answers it's a good ides to first peruse duplicate links before posting an answer to a five year old question!
      $endgroup$
      – Bill Dubuque
      Dec 31 '16 at 2:03






      $begingroup$
      This is precisely the homogenization $(a^n-1to a^n-b^n)$ of a proof in the 5-year-old duplicate thread linked in Yuan's comment on the question. To avoid posting such duplicate answers it's a good ides to first peruse duplicate links before posting an answer to a five year old question!
      $endgroup$
      – Bill Dubuque
      Dec 31 '16 at 2:03














      $begingroup$
      Update: actually this homegenized version was posted 5 months prior in this answer.. There are probably older dupes too since this is a FAQ. Posting the link in case anyone decides to organize.
      $endgroup$
      – Bill Dubuque
      Jul 13 '17 at 20:44






      $begingroup$
      Update: actually this homegenized version was posted 5 months prior in this answer.. There are probably older dupes too since this is a FAQ. Posting the link in case anyone decides to organize.
      $endgroup$
      – Bill Dubuque
      Jul 13 '17 at 20:44














      $begingroup$
      I didn't understand why if gcd$(a,b) =1$ then gcd$(b, d) =1$? and why $left(ab^{-1}right)^mequiv left(ab^{-1}right)^nequiv 1pmod{d}$?
      $endgroup$
      – Vmimi
      Dec 2 '18 at 23:48




      $begingroup$
      I didn't understand why if gcd$(a,b) =1$ then gcd$(b, d) =1$? and why $left(ab^{-1}right)^mequiv left(ab^{-1}right)^nequiv 1pmod{d}$?
      $endgroup$
      – Vmimi
      Dec 2 '18 at 23:48











      8












      $begingroup$

      More generally, if $a,b,m,ninmathbb Z_{ge 1}$, $a>b$ and $(a,b)=1$ (as usual, $(a,b)$ denotes $gcd(a,b)$), then $$(a^m-b^m,a^n-b^n)=a^{(m,n)}-b^{(m,n)}$$



      Proof: Use $,x^k-y^k=(x-y)(x^{k-1}+x^{k-2}y+cdots+xy^{k-2}+x^{k-1}),$



      and use $nmid a,biff nmid (a,b)$ to prove:



      $a^{(m,n)}-b^{(m,n)}mid a^m-b^m,, a^n-b^niff$



      $a^{(m,n)}-b^{(m,n)}mid (a^m-b^m,a^n-b^n)=: d (1)$



      $a^mequiv b^m,, a^nequiv b^n$ mod $d$ by definition of $d$.



      Bezout's lemma gives $,mx+ny=(m,n),$ for some $x,yinBbb Z$.



      $(a,b)=1iff (a,d)=(b,d)=1$, so $a^{mx},b^{ny}$ mod $d$ exist (notice $x,y$ can be negative).



      $a^{mx}equiv b^{mx}$, $a^{ny}equiv b^{ny}$ mod $d$.



      $a^{(m,n)}equiv a^{mx}a^{ny}equiv b^{mx}b^{ny}equiv b^{(m,n)}pmod{! d} (2)$



      $(1)(2),Rightarrow, a^{(m,n)}-b^{(m,n)}=d$






      share|cite|improve this answer











      $endgroup$













      • $begingroup$
        What is $d$? I don't understand why gcd$(b, d)=1 $ and why do you need that to be true?
        $endgroup$
        – Vmimi
        Nov 30 '18 at 0:22


















      8












      $begingroup$

      More generally, if $a,b,m,ninmathbb Z_{ge 1}$, $a>b$ and $(a,b)=1$ (as usual, $(a,b)$ denotes $gcd(a,b)$), then $$(a^m-b^m,a^n-b^n)=a^{(m,n)}-b^{(m,n)}$$



      Proof: Use $,x^k-y^k=(x-y)(x^{k-1}+x^{k-2}y+cdots+xy^{k-2}+x^{k-1}),$



      and use $nmid a,biff nmid (a,b)$ to prove:



      $a^{(m,n)}-b^{(m,n)}mid a^m-b^m,, a^n-b^niff$



      $a^{(m,n)}-b^{(m,n)}mid (a^m-b^m,a^n-b^n)=: d (1)$



      $a^mequiv b^m,, a^nequiv b^n$ mod $d$ by definition of $d$.



      Bezout's lemma gives $,mx+ny=(m,n),$ for some $x,yinBbb Z$.



      $(a,b)=1iff (a,d)=(b,d)=1$, so $a^{mx},b^{ny}$ mod $d$ exist (notice $x,y$ can be negative).



      $a^{mx}equiv b^{mx}$, $a^{ny}equiv b^{ny}$ mod $d$.



      $a^{(m,n)}equiv a^{mx}a^{ny}equiv b^{mx}b^{ny}equiv b^{(m,n)}pmod{! d} (2)$



      $(1)(2),Rightarrow, a^{(m,n)}-b^{(m,n)}=d$






      share|cite|improve this answer











      $endgroup$













      • $begingroup$
        What is $d$? I don't understand why gcd$(b, d)=1 $ and why do you need that to be true?
        $endgroup$
        – Vmimi
        Nov 30 '18 at 0:22
















      8












      8








      8





      $begingroup$

      More generally, if $a,b,m,ninmathbb Z_{ge 1}$, $a>b$ and $(a,b)=1$ (as usual, $(a,b)$ denotes $gcd(a,b)$), then $$(a^m-b^m,a^n-b^n)=a^{(m,n)}-b^{(m,n)}$$



      Proof: Use $,x^k-y^k=(x-y)(x^{k-1}+x^{k-2}y+cdots+xy^{k-2}+x^{k-1}),$



      and use $nmid a,biff nmid (a,b)$ to prove:



      $a^{(m,n)}-b^{(m,n)}mid a^m-b^m,, a^n-b^niff$



      $a^{(m,n)}-b^{(m,n)}mid (a^m-b^m,a^n-b^n)=: d (1)$



      $a^mequiv b^m,, a^nequiv b^n$ mod $d$ by definition of $d$.



      Bezout's lemma gives $,mx+ny=(m,n),$ for some $x,yinBbb Z$.



      $(a,b)=1iff (a,d)=(b,d)=1$, so $a^{mx},b^{ny}$ mod $d$ exist (notice $x,y$ can be negative).



      $a^{mx}equiv b^{mx}$, $a^{ny}equiv b^{ny}$ mod $d$.



      $a^{(m,n)}equiv a^{mx}a^{ny}equiv b^{mx}b^{ny}equiv b^{(m,n)}pmod{! d} (2)$



      $(1)(2),Rightarrow, a^{(m,n)}-b^{(m,n)}=d$






      share|cite|improve this answer











      $endgroup$



      More generally, if $a,b,m,ninmathbb Z_{ge 1}$, $a>b$ and $(a,b)=1$ (as usual, $(a,b)$ denotes $gcd(a,b)$), then $$(a^m-b^m,a^n-b^n)=a^{(m,n)}-b^{(m,n)}$$



      Proof: Use $,x^k-y^k=(x-y)(x^{k-1}+x^{k-2}y+cdots+xy^{k-2}+x^{k-1}),$



      and use $nmid a,biff nmid (a,b)$ to prove:



      $a^{(m,n)}-b^{(m,n)}mid a^m-b^m,, a^n-b^niff$



      $a^{(m,n)}-b^{(m,n)}mid (a^m-b^m,a^n-b^n)=: d (1)$



      $a^mequiv b^m,, a^nequiv b^n$ mod $d$ by definition of $d$.



      Bezout's lemma gives $,mx+ny=(m,n),$ for some $x,yinBbb Z$.



      $(a,b)=1iff (a,d)=(b,d)=1$, so $a^{mx},b^{ny}$ mod $d$ exist (notice $x,y$ can be negative).



      $a^{mx}equiv b^{mx}$, $a^{ny}equiv b^{ny}$ mod $d$.



      $a^{(m,n)}equiv a^{mx}a^{ny}equiv b^{mx}b^{ny}equiv b^{(m,n)}pmod{! d} (2)$



      $(1)(2),Rightarrow, a^{(m,n)}-b^{(m,n)}=d$







      share|cite|improve this answer














      share|cite|improve this answer



      share|cite|improve this answer








      edited Jan 5 '18 at 23:28


























      community wiki





      8 revs
      user26486













      • $begingroup$
        What is $d$? I don't understand why gcd$(b, d)=1 $ and why do you need that to be true?
        $endgroup$
        – Vmimi
        Nov 30 '18 at 0:22




















      • $begingroup$
        What is $d$? I don't understand why gcd$(b, d)=1 $ and why do you need that to be true?
        $endgroup$
        – Vmimi
        Nov 30 '18 at 0:22


















      $begingroup$
      What is $d$? I don't understand why gcd$(b, d)=1 $ and why do you need that to be true?
      $endgroup$
      – Vmimi
      Nov 30 '18 at 0:22






      $begingroup$
      What is $d$? I don't understand why gcd$(b, d)=1 $ and why do you need that to be true?
      $endgroup$
      – Vmimi
      Nov 30 '18 at 0:22













      5












      $begingroup$

      Let
      $$gcd(a^n - 1, a^m - 1) = t$$
      then
      $$a^n equiv 1 ,big(text{ mod } tbig),quadtext{and}quad,a^m equiv 1 ,big(text{ mod } tbig)$$
      And thus
      $$a^{nx + my} equiv 1, big(text{ mod } tbig)$$
      $forall,x,,yin mathbb{Z}$



      According to the Extended Euclidean algorithm, we have
      $$nx + my =gcd(n,m)$$
      This follows
      $$a^{nx + my} equiv 1 ,big(text{ mod } tbig) = a^{gcd(n,m)} equiv 1 big(text{ mod } tbig)impliesbig( a^{gcd(n,m)} - 1big) big| t$$



      Therefore
      $$a^{gcd(m,n)}-1, =gcd(a^m-1, a^n-1) $$






      share|cite|improve this answer











      $endgroup$


















        5












        $begingroup$

        Let
        $$gcd(a^n - 1, a^m - 1) = t$$
        then
        $$a^n equiv 1 ,big(text{ mod } tbig),quadtext{and}quad,a^m equiv 1 ,big(text{ mod } tbig)$$
        And thus
        $$a^{nx + my} equiv 1, big(text{ mod } tbig)$$
        $forall,x,,yin mathbb{Z}$



        According to the Extended Euclidean algorithm, we have
        $$nx + my =gcd(n,m)$$
        This follows
        $$a^{nx + my} equiv 1 ,big(text{ mod } tbig) = a^{gcd(n,m)} equiv 1 big(text{ mod } tbig)impliesbig( a^{gcd(n,m)} - 1big) big| t$$



        Therefore
        $$a^{gcd(m,n)}-1, =gcd(a^m-1, a^n-1) $$






        share|cite|improve this answer











        $endgroup$
















          5












          5








          5





          $begingroup$

          Let
          $$gcd(a^n - 1, a^m - 1) = t$$
          then
          $$a^n equiv 1 ,big(text{ mod } tbig),quadtext{and}quad,a^m equiv 1 ,big(text{ mod } tbig)$$
          And thus
          $$a^{nx + my} equiv 1, big(text{ mod } tbig)$$
          $forall,x,,yin mathbb{Z}$



          According to the Extended Euclidean algorithm, we have
          $$nx + my =gcd(n,m)$$
          This follows
          $$a^{nx + my} equiv 1 ,big(text{ mod } tbig) = a^{gcd(n,m)} equiv 1 big(text{ mod } tbig)impliesbig( a^{gcd(n,m)} - 1big) big| t$$



          Therefore
          $$a^{gcd(m,n)}-1, =gcd(a^m-1, a^n-1) $$






          share|cite|improve this answer











          $endgroup$



          Let
          $$gcd(a^n - 1, a^m - 1) = t$$
          then
          $$a^n equiv 1 ,big(text{ mod } tbig),quadtext{and}quad,a^m equiv 1 ,big(text{ mod } tbig)$$
          And thus
          $$a^{nx + my} equiv 1, big(text{ mod } tbig)$$
          $forall,x,,yin mathbb{Z}$



          According to the Extended Euclidean algorithm, we have
          $$nx + my =gcd(n,m)$$
          This follows
          $$a^{nx + my} equiv 1 ,big(text{ mod } tbig) = a^{gcd(n,m)} equiv 1 big(text{ mod } tbig)impliesbig( a^{gcd(n,m)} - 1big) big| t$$



          Therefore
          $$a^{gcd(m,n)}-1, =gcd(a^m-1, a^n-1) $$







          share|cite|improve this answer














          share|cite|improve this answer



          share|cite|improve this answer








          edited Dec 1 '17 at 14:40

























          answered Dec 1 '17 at 14:34









          Darío A. GutiérrezDarío A. Gutiérrez

          2,66441530




          2,66441530























              3












              $begingroup$

              Written for a duplicate question, this may be a bit more elementary than the other answers here, so I will post it:





              If $g=(a,b)$ and $G=left(p^a-1,p^b-1right)$, then
              $$
              left(p^g-1right)sum_{k=0}^{frac ag-1}p^{kg}=p^a-1
              $$
              and
              $$
              left(p^g-1right)sum_{k=0}^{frac bg-1}p^{kg}=p^b-1
              $$
              Thus, we have that
              $$
              left.p^g-1,middle|,Gright.
              $$





              For $xge0$,
              $$
              left(p^a-1right)sum_{k=0}^{x-1}p^{ak}=p^{ax}-1
              $$
              Therefore, we have that
              $$
              left.G,middle|,p^{ax}-1right.
              $$
              If $left.G,middle|,p^{ax-(b-1)y}-1right.$, then
              $$
              left(p^{ax-(b-1)y}-1right)-p^{ax-by}left(p^b-1right)=p^{ax-by}-1
              $$
              Therefore, for any $x,yge0$ so that $ax-byge0$,
              $$
              left.G,middle|,p^{ax-by}-1right.
              $$
              which means that
              $$
              left.G,middle|,p^g-1right.
              $$





              Putting all this together gives
              $$
              G=p^g-1
              $$






              share|cite|improve this answer











              $endgroup$


















                3












                $begingroup$

                Written for a duplicate question, this may be a bit more elementary than the other answers here, so I will post it:





                If $g=(a,b)$ and $G=left(p^a-1,p^b-1right)$, then
                $$
                left(p^g-1right)sum_{k=0}^{frac ag-1}p^{kg}=p^a-1
                $$
                and
                $$
                left(p^g-1right)sum_{k=0}^{frac bg-1}p^{kg}=p^b-1
                $$
                Thus, we have that
                $$
                left.p^g-1,middle|,Gright.
                $$





                For $xge0$,
                $$
                left(p^a-1right)sum_{k=0}^{x-1}p^{ak}=p^{ax}-1
                $$
                Therefore, we have that
                $$
                left.G,middle|,p^{ax}-1right.
                $$
                If $left.G,middle|,p^{ax-(b-1)y}-1right.$, then
                $$
                left(p^{ax-(b-1)y}-1right)-p^{ax-by}left(p^b-1right)=p^{ax-by}-1
                $$
                Therefore, for any $x,yge0$ so that $ax-byge0$,
                $$
                left.G,middle|,p^{ax-by}-1right.
                $$
                which means that
                $$
                left.G,middle|,p^g-1right.
                $$





                Putting all this together gives
                $$
                G=p^g-1
                $$






                share|cite|improve this answer











                $endgroup$
















                  3












                  3








                  3





                  $begingroup$

                  Written for a duplicate question, this may be a bit more elementary than the other answers here, so I will post it:





                  If $g=(a,b)$ and $G=left(p^a-1,p^b-1right)$, then
                  $$
                  left(p^g-1right)sum_{k=0}^{frac ag-1}p^{kg}=p^a-1
                  $$
                  and
                  $$
                  left(p^g-1right)sum_{k=0}^{frac bg-1}p^{kg}=p^b-1
                  $$
                  Thus, we have that
                  $$
                  left.p^g-1,middle|,Gright.
                  $$





                  For $xge0$,
                  $$
                  left(p^a-1right)sum_{k=0}^{x-1}p^{ak}=p^{ax}-1
                  $$
                  Therefore, we have that
                  $$
                  left.G,middle|,p^{ax}-1right.
                  $$
                  If $left.G,middle|,p^{ax-(b-1)y}-1right.$, then
                  $$
                  left(p^{ax-(b-1)y}-1right)-p^{ax-by}left(p^b-1right)=p^{ax-by}-1
                  $$
                  Therefore, for any $x,yge0$ so that $ax-byge0$,
                  $$
                  left.G,middle|,p^{ax-by}-1right.
                  $$
                  which means that
                  $$
                  left.G,middle|,p^g-1right.
                  $$





                  Putting all this together gives
                  $$
                  G=p^g-1
                  $$






                  share|cite|improve this answer











                  $endgroup$



                  Written for a duplicate question, this may be a bit more elementary than the other answers here, so I will post it:





                  If $g=(a,b)$ and $G=left(p^a-1,p^b-1right)$, then
                  $$
                  left(p^g-1right)sum_{k=0}^{frac ag-1}p^{kg}=p^a-1
                  $$
                  and
                  $$
                  left(p^g-1right)sum_{k=0}^{frac bg-1}p^{kg}=p^b-1
                  $$
                  Thus, we have that
                  $$
                  left.p^g-1,middle|,Gright.
                  $$





                  For $xge0$,
                  $$
                  left(p^a-1right)sum_{k=0}^{x-1}p^{ak}=p^{ax}-1
                  $$
                  Therefore, we have that
                  $$
                  left.G,middle|,p^{ax}-1right.
                  $$
                  If $left.G,middle|,p^{ax-(b-1)y}-1right.$, then
                  $$
                  left(p^{ax-(b-1)y}-1right)-p^{ax-by}left(p^b-1right)=p^{ax-by}-1
                  $$
                  Therefore, for any $x,yge0$ so that $ax-byge0$,
                  $$
                  left.G,middle|,p^{ax-by}-1right.
                  $$
                  which means that
                  $$
                  left.G,middle|,p^g-1right.
                  $$





                  Putting all this together gives
                  $$
                  G=p^g-1
                  $$







                  share|cite|improve this answer














                  share|cite|improve this answer



                  share|cite|improve this answer








                  edited Mar 15 '18 at 14:32

























                  answered Mar 15 '18 at 14:11









                  robjohnrobjohn

                  269k27311638




                  269k27311638






























                      draft saved

                      draft discarded




















































                      Thanks for contributing an answer to Mathematics Stack Exchange!


                      • Please be sure to answer the question. Provide details and share your research!

                      But avoid



                      • Asking for help, clarification, or responding to other answers.

                      • Making statements based on opinion; back them up with references or personal experience.


                      Use MathJax to format equations. MathJax reference.


                      To learn more, see our tips on writing great answers.




                      draft saved


                      draft discarded














                      StackExchange.ready(
                      function () {
                      StackExchange.openid.initPostLogin('.new-post-login', 'https%3a%2f%2fmath.stackexchange.com%2fquestions%2f7473%2fprove-that-gcdan-1-am-1-a-gcdn-m-1%23new-answer', 'question_page');
                      }
                      );

                      Post as a guest















                      Required, but never shown





















































                      Required, but never shown














                      Required, but never shown












                      Required, but never shown







                      Required, but never shown

































                      Required, but never shown














                      Required, but never shown












                      Required, but never shown







                      Required, but never shown







                      Popular posts from this blog

                      Magento 2 - Add success message with knockout Planned maintenance scheduled April 23, 2019 at 23:30 UTC (7:30pm US/Eastern) Announcing the arrival of Valued Associate #679: Cesar Manara Unicorn Meta Zoo #1: Why another podcast?Success / Error message on ajax request$.widget is not a function when loading a homepage after add custom jQuery on custom themeHow can bind jQuery to current document in Magento 2 When template load by ajaxRedirect page using plugin in Magento 2Magento 2 - Update quantity and totals of cart page without page reload?Magento 2: Quote data not loaded on knockout checkoutMagento 2 : I need to change add to cart success message after adding product into cart through pluginMagento 2.2.5 How to add additional products to cart from new checkout step?Magento 2 Add error/success message with knockoutCan't validate Post Code on checkout page

                      Fil:Tokke komm.svg

                      Where did Arya get these scars? Unicorn Meta Zoo #1: Why another podcast? Announcing the arrival of Valued Associate #679: Cesar Manara Favourite questions and answers from the 1st quarter of 2019Why did Arya refuse to end it?Has the pronunciation of Arya Stark's name changed?Has Arya forgiven people?Why did Arya Stark lose her vision?Why can Arya still use the faces?Has the Narrow Sea become narrower?Does Arya Stark know how to make poisons outside of the House of Black and White?Why did Nymeria leave Arya?Why did Arya not kill the Lannister soldiers she encountered in the Riverlands?What is the current canonical age of Sansa, Bran and Arya Stark?